Merge remote branch 'public/master'
[course.git] / latex / problems / Serway_and_Jewett_4 / problem20.41.tex
1 \begin{problem*}{20.41}
2 Four capacitors are connected as shown in Figure P20.41.
3 \Part{a} Find the equivalent capacitance between points $a$ and $b$.
4 \Part{b} Calculate the charge on each capacitor, taking
5  $\Delta V_{ab} = 15.0\U{V}$
6 \end{problem*} % problem 20.41
7
8 \begin{solution}
9 \Part{a}
10 First consider the top two capacitors, $C_1 = 15.0\U{$\mu$F}$ and
11  $C_2 = 3.00\U{$\mu$F}$.
12 They are in series, so the effective capacitance of the top line is
13  given by
14 \begin{equation}
15  C_t = \left(\frac{1}{C_1} + \frac{1}{C_2}\right)^{-1}
16      = 2.50\U{$\mu$F}
17 \end{equation}
18
19 We can find the effective capacitance of the box, because $C_t$ is
20 in parallel with $C_3 = 6.00\U{$\mu$F}$.
21 \begin{equation}
22  C_b = C_t + C_3 = 8.50\U{$\mu$C}
23 \end{equation}
24
25 We can find the total equivalent capacitance, because $C_b$ is in
26 series with $C_4 = 20.0\U{$\mu$F}$.
27 \begin{equation}
28  C_{eq} = \left(\frac{1}{C_b} + \frac{1}{C_4}\right)^{-1}
29         = \ans{ 5.96\U{$\mu$F}}
30 \end{equation}
31
32 \Part{b}
33 Working backwards to find the charges, using $Q = CV$, we have
34 \begin{equation}
35  Q_4 = Q_b = C_{eq} V_{ab} = \ans{89.5\U{$\mu$C}}
36 \end{equation}
37 So the voltage across the box is
38 \begin{equation}
39  V_b = \frac{Q_b}{C_b} = 10.5\U{V}
40 \end{equation}
41 So
42 \begin{equation}
43  Q_3 = C_3 V_b = \ans{63.2\U{$\mu$C}}
44 \end{equation}
45 and
46 \begin{equation}
47  Q_1 = Q_2 = C_t V_b = \ans{26.3\U{$\mu$C}}
48 \end{equation}
49 \end{solution}